[Math] $\{f_n\}$ is uniformly integrable if and only if $\sup_n \int |f_n|\,d\mu < \infty$ and $\{f_n\}$ is uniformly absolutely continuous

integrationmeasure-theoryprobability theoryreal-analysissequences-and-series

Let $(X, \mathcal{A}, \mu)$ be a measure space. A family of measurable functions $\{f_n\}$ is uniformly integrable if given $\epsilon$ there exists $M$ such that$$\int_{\{x : |f_n(x)| > M\}} |f_n(x)|\,d\mu < \epsilon$$for each $n$. The sequence is uniformly absolutely continuous if given $\epsilon$ there exists $\delta$ such that$$\left|\int_A f_n\,d\mu\right| < \epsilon$$for each $n$ if $\mu(A) < \delta$.

Suppose $\mu$ is a finite measure. How do I see that $\{f_n\}$ is uniformly integrable if and only if $\sup_n \int |f_n|\,d\mu < \infty$ and $\{f_n\}$ is uniformly absolutely continuous?

Best Answer

Assume that $\{f_n\}$ is uniformly absolutely integrable. Let $\varepsilon > 0$. Now \begin{align} \int_X |f_n| = \int_{X \cap \{f_n \geq M_\varepsilon\}} |f_n| + \int_{X \cap \{f_n < M_\varepsilon\}} |f_n| \leq \varepsilon + M_\varepsilon \mu(X) \end{align} for all $n$. Thus the supremum over $n$ is finite.

To get uniform absolute continuity, notice that \begin{align} \left| \int_A f_n \right| & \leq \int_{A \cap \{f_n \geq M_\varepsilon\}} |f_n| + \int_{A \cap \{f_n < M_\varepsilon\}} |f_n| \\ & \leq \varepsilon + M_\varepsilon \mu(A) \end{align} for all $n$. Now choose $\delta < \varepsilon/M_\varepsilon$.

Now assume $\sup_n \|f_n\|_1 < \infty$ and the uniform abs. continuity. Let $\varepsilon > 0$ and let $\delta > 0$ be such that $\mu(A) < \delta$ implies $ |\int_A f_n| < \varepsilon$ for all $n$. Since $\int|f_n| < \infty$, we have \begin{align} \lim_{M \to \infty} \mu \{ |f_n| > M \} = 0\,. \end{align} Thus we may choose $M_n$ so large that $\mu\{ |f_n| > M_n \} < \delta$. Now \begin{align} \int_{|f_n| > M_n } |f_n| &= \int_{f_n > M_n} f_n + \int_{f_n < -M_n} (-f_n) \\ &= \left| \int_{f_n > M_n} f_n \right| + \left| \int_{f_n < -M_n} f_n \right| \\ &< \varepsilon + \varepsilon \end{align} for all $n$ since the sets over which we integrate have measure less than $\delta$

Related Question